Đến nội dung

babylearnmath294 nội dung

Có 38 mục bởi babylearnmath294 (Tìm giới hạn từ 07-06-2020)



Sắp theo                Sắp xếp  

#51758 Xưa như trái dưa

Đã gửi bởi babylearnmath294 on 05-01-2006 - 22:58 trong Bất đẳng thức - Cực trị

bai nay trong mot quyen sach bat dang thuc cua Nguyen Duc Tan



#46617 Crux Mathematicorum, problem 2645, Hojoo Lee

Đã gửi bởi babylearnmath294 on 10-12-2005 - 10:20 trong Bất đẳng thức - Cực trị

chú hungkhtn ,cho tôi hỏi phuơng pháp SOS là phương pháp gì vậy???



#46614 PT

Đã gửi bởi babylearnmath294 on 10-12-2005 - 10:06 trong Phương trình - hệ phương trình - bất phương trình

bài này quá dễ,đề nghị vanchanh ko nên spam bài như vậy



#44680 Hay lắm đó

Đã gửi bởi babylearnmath294 on 29-11-2005 - 18:42 trong Phương trình - hệ phương trình - bất phương trình

THANIQ làm sai rồi



#44344 bài hay

Đã gửi bởi babylearnmath294 on 27-11-2005 - 21:03 trong Phương trình - hệ phương trình - bất phương trình

Bài này không khó,đúng như cách của HaiDang nói:

Sử dụng BDT

(Với cũng thỏa mãn)
...



#44199 1 Bài không mới

Đã gửi bởi babylearnmath294 on 27-11-2005 - 07:35 trong Bất đẳng thức - Cực trị

Nói như bạn cũng đúng,thì tui cũng đã giải được đâu!!!!
Có lẽ bài này phải cho vào box 'chưa có lời giải mới đúng'.



#44170 1 Bài không mới

Đã gửi bởi babylearnmath294 on 26-11-2005 - 21:38 trong Bất đẳng thức - Cực trị

Tôi hiểu ý bạn rồi.Nhưng nói như vậy còn mơ hồ quá.Vậy tức là bài toán tổng quát là không thể làm được với nghiệm quá lẻ phải không???
Vậy nếu giới hạn cho số mũ http://dientuvietnam...n/mimetex.cgi?y là số nguyên dương
thì vẫn không làm đươc phải không ???



#44063 1 Bài không mới

Đã gửi bởi babylearnmath294 on 26-11-2005 - 12:19 trong Bất đẳng thức - Cực trị

Các bác ơi,chẳng nhẽ tổng quát kiểu này là tham lam quá hay sao mà chẳng ai giúp em vậy??????????



#44000 xin moi (hay lam)

Đã gửi bởi babylearnmath294 on 26-11-2005 - 07:30 trong Bất đẳng thức - Cực trị

Cách giải của Mr Math giống cách của em,thế còn anh Hùng thì sao,đưa 1 cách khác lên đi anh???



#43790 xin moi (hay lam)

Đã gửi bởi babylearnmath294 on 25-11-2005 - 10:22 trong Bất đẳng thức - Cực trị

Cách giải dung BUNHIa thì nói làm gì hả các anh!!!
Em nghĩ rằng bài này có 1 cách hay hơn,vấn đề là chưa tìm ra thôi.



#43775 xin moi (hay lam)

Đã gửi bởi babylearnmath294 on 25-11-2005 - 09:38 trong Bất đẳng thức - Cực trị

Anh hungkhtn ơi,liệu anh có thể đề ra 1 cách giải cho bọn em cùng xem không,em cũng háo hức lắm. Cách làm của em tuy không trâu bò lắm nhưng thực sự thì không thể cho là đẹp được.



#43759 Bài toán đã cũ

Đã gửi bởi babylearnmath294 on 24-11-2005 - 23:56 trong Bất đẳng thức - Cực trị

Sorry,nhầm chút xíu,bài này còn dễ hơn http://dientuvietnam...imetex.cgi?COSI thì như sau:

Dễ thấy http://dientuvietnam...mimetex.cgi?MAX của:

....Vậy là xong rồi.!



#43296 xin moi (hay lam)

Đã gửi bởi babylearnmath294 on 21-11-2005 - 22:19 trong Bất đẳng thức - Cực trị

Bài này thực sự là ko thuần nhất ở cả 2 vế của BDT,việc đặt k=1 là hoàn toàn ko được.Nên xem lại trong box chuyên đề để hiểu rõ hơn về hàm thuần nhất.
Và bài toán này nếu muốn 1 lời giải đẹp thì vẫn không hề đơn giản!!!!!!!!!!



#43189 xin moi (hay lam)

Đã gửi bởi babylearnmath294 on 21-11-2005 - 09:43 trong Bất đẳng thức - Cực trị

Tôi thấy bạn đã nhầm to khi cho rằng hàm F và G ở 2 vế của BDT là thuần nhất. Bạn thử xem lại xem: cứ cho là nó thuần nhất bậc 1 thì vì có 1 ở mẫu số nên khi thay a,b,c thành ta,tb,tc ,nó
trở thành 1 BDT khác rồi và F(ta,tb,tc) # tF(a,b,c)
Do đó việc giả sử abc=1 là hoàn toàn không được.



#43155 xin moi (hay lam)

Đã gửi bởi babylearnmath294 on 20-11-2005 - 22:15 trong Bất đẳng thức - Cực trị

Nếu nói như hochoi2005 thì chẳng nhẽ bài BDT nào có các biến đối xứng cũng đều có thể đặt K=1 được sao????
Như vậy vô tình cậu đã làm nó đơn giản đi rất nhiều!!!
Có lẽ đó là do bạn chưa hiểu hết về hàm thuần nhất.
Bạn nên xem lại các bài viết chuyên đề để hiểu rõ hơn.



#43055 xin moi (hay lam)

Đã gửi bởi babylearnmath294 on 20-11-2005 - 11:50 trong Bất đẳng thức - Cực trị

À,các anh có thể giải thích cho em rõ hơn về BDT thuần nhất được ko???
Và khi nào thì ta có thể giả sử tích hoặc tổng cua chúng là 1 giá trị xác định (1,2,3..).Em thấy đó là 1 phần rất hay của BDT.Cảm ơn trước!!!!!



#43047 xin moi (hay lam)

Đã gửi bởi babylearnmath294 on 20-11-2005 - 11:27 trong Bất đẳng thức - Cực trị

Tui cũng nghĩ là việc đặt abc=1 là chưa hợp lý,vì ở mẫu có 1 nên cần xem
xét lại.Tôi nghĩ có lẽ nên đặt abc=K ,sau đó biểu diễn 1 theo abc và K,
rồi COSI theo cách của cậu xem,có thể sẽ được đấy.



#42876 một bài gần cũ

Đã gửi bởi babylearnmath294 on 19-11-2005 - 13:28 trong Bất đẳng thức - Cực trị

Thì ra là thế!!!
Nhưng cái bài ở dưới thì tất nhiên là ko dễ chút nào,nếu muốn có 1 cách đẹp thì càng khó hơn.



#42871 xin moi (hay lam)

Đã gửi bởi babylearnmath294 on 19-11-2005 - 12:59 trong Bất đẳng thức - Cực trị

Nếu bạn có 1 cách hay như vậy thì đưa lên để mọi người cùng xem.
Quả thực bài này để có 1 cách giải đẹp mắt cũng ko phải là dễ.



#42717 1 Bài không mới

Đã gửi bởi babylearnmath294 on 18-11-2005 - 12:24 trong Bất đẳng thức - Cực trị

Em mới chỉ giải được 1 dạng của bài trên và đã từng đưa lên diễn đàn,đó là:
http://dientuvietnam...tex.cgi?>2,chịu thua!!!
Và khi đó,dường như chỉ có đ/k tổng các biến http://dientuvietnam.../mimetex.cgi?=Z là chưa đủ.

Mong mọi người góp ý cho em!!!!!!!!Thanks a lot!!!!!!!!!!!!



#42709 1 Bài không mới

Đã gửi bởi babylearnmath294 on 18-11-2005 - 11:44 trong Bất đẳng thức - Cực trị

Cho http://dientuvietnam...tex.cgi?a,b,c>0http://dientuvietnam...mimetex.cgi?MIN của :


http://dientuvietnam...mimetex.cgi?MIN của:

http://dientuvietnam...mimetex.cgi?MIN với mọi http://dientuvietnam...metex.cgi?Z????
Mong các bạn giúp đỡ.



#42652 1 bài ko khó

Đã gửi bởi babylearnmath294 on 17-11-2005 - 23:04 trong Bất đẳng thức - Cực trị

Vậy giả sử khi ta giải hệ PT đó mà vô nghiệm thì có thể khẳng định hàm số đó không có cực trị không nhỉ??????
Và tôi nghĩ đó cũng là 1 hạn chế của cách giải này.
Theo bạn thì sao????



#42374 1 bài ko khó

Đã gửi bởi babylearnmath294 on 15-11-2005 - 23:15 trong Bất đẳng thức - Cực trị

Lạ quá,sao tui cũng làm 1 cách tương tự như vậy mà sao nghiệm ra rất lẻ
Mời bạn xem thử:
COSI ta có
a^2 + x^2>= 2ax

b^3 + y^3+ y^3>=3yyb

c^3 + y^3+ y^3>=3yyc

Ta đoán rằng b=c ,lại có a+b+c=1 nên quy về việc giải hệ sau:

a+2b=1
2a=3b ^2
Ra nghiệm rất lẻ
2 cách làm này có gì khác nhau ko nhỉ????????

Xin lỗi ,quả thực tôi ko hiểu cách COSI của bạn cho lắm,có thể giải thích rõ hơn ko (cũng có thể do tôi quá kém chăng????).



#42351 1 bài ko khó

Đã gửi bởi babylearnmath294 on 15-11-2005 - 18:47 trong Bất đẳng thức - Cực trị

Nhờ mọi người làm giúp bài này:

Cho a,b,c>0 a+b+c=1
Tim MIN
P=a^2 + b^3 + c^3



#42340 một bài gần cũ

Đã gửi bởi babylearnmath294 on 15-11-2005 - 18:14 trong Bất đẳng thức - Cực trị

Sorry,nhầm chút xíu, mà cũng vậy thôi,sửa lại chút là được:

abc>=1 nên xyz>=1 nên


xy(x+y+z)=xyz(x+y+z)/z>=(x+y+z)/z


Được chưa hả "bác"!!!!!!!!

Mà bài của bác ko mạnh bằng bài trước đó.